If \(p = \begin{pmatrix} 2 \\ -2 \end{pmatrix} \) and \(q = \begin{pmatrix} 3 \\ 4 \end{pmatrix}\), find \(|q - \frac{1}{2}p|\).

A.

\(2\sqrt{2}\)

B.

\(\sqrt{13}\)

C.

\(5\)

D.

\(\sqrt{29}\)

Correct answer is D

\(p = \begin{pmatrix} 2 \\ -2 \end{pmatrix} , q = \begin{pmatrix} 3 \\ 4 \end{pmatrix}\)

\(\frac{1}{2}p = \begin{pmatrix} 1 \\ -1 \end{pmatrix}\)

\(q - \frac{1}{2}p = \begin{pmatrix} 3 \\ 4 \end{pmatrix} - \begin{pmatrix} 1 \\ -1 \end{pmatrix} = \begin{pmatrix} 2 \\ 5 \end{pmatrix}\)

\(|q - \frac{1}{2}p| = \sqrt{2^{2} + 5^{2}} = \sqrt{29}\)